0 Daumen
326 Aufrufe

Ermitteln Sie eine Rekursionsformel für die Folge \((I_n)_{n\in \mathbb{N}}\) mit

$$ I_n:=\int\limits \frac{1}{(1+x^2)^n}\text{ dx} $$

Danke im Voraus

Avatar von

Ist die Aufgabe so vollständig und richtig notiert?

3 Antworten

0 Daumen

Aloha :)

Oha, hier liegt der Duft nach partieller Integration in der Luft:$$I_n=\int\frac{dx}{(x^2+1)^n}=\int\underbrace{1}_{=u'}\cdot\underbrace{\frac{1}{(x^2+1)^n}}_{=v}\,dx=\underbrace{x}_{=u}\cdot\underbrace{\frac{1}{(x^2+1)^n}}_{=v}-\int\underbrace{x}_{=u}\cdot\underbrace{\frac{-n\cdot2x}{(x^2+1)^{n+1}}}_{=v'}\,dx$$$$\phantom{I_n}=\frac{x}{(x^2+1)^n}+2n\cdot\int\frac{x^2}{(x^2+1)^{n+1}}\,dx=\frac{x}{(x^2+1)^n}+2n\int\frac{(x^2+1)-1}{(x^2+1)^{n+1}}dx$$$$\phantom{I_n}=\frac{x}{(x^2+1)^n}+2n\int\left(\frac{(x^2+1)}{(x^2+1)^{n+1}}-\frac{1}{(x^2+1)^{n+1}}\right)dx$$$$\phantom{I_n}=\frac{x}{(x^2+1)^n}+2n\int\left(\frac{1}{(x^2+1)^{n}}-\frac{1}{(x^2+1)^{n+1}}\right)dx$$$$\phantom{I_n}=\frac{x}{(x^2+1)^n}+2n\int\frac{dx}{(x^2+1)^n}-2n\int\frac{dx}{(x^2+1)^{n+1}}$$$$\phantom{I_n}=\frac{x}{(x^2+1)^n}+2nI_n-2nI_{n+1}$$Das stellen wir nach \(I_{n+1}\) um:$$2nI_{n+1}=\frac{x}{(x^2+1)^n}+2nI_n-I_n=\frac{x}{(x^2+1)^n}+(2n-1)I_n$$$$I_{n+1}=\frac{1}{2n}\,\frac{x}{(x^2+1)^n}+\frac{2n-1}{2n}\,I_n\quad;\quad I_1=\int\frac{dx}{x^2+1}=\arctan(x)$$

Avatar von 148 k 🚀

Frage: Im letzten Integral auf der ersten Zeile steht im Zähler ein x^2. Wo ist das geblieben?

Ist nicht I1 = arctan(x)  ?

Danke euch, ich hatte mich verrechnet... Habe es korrigiert ;)

Besser auch noch das Io rausnehmen.

Ich habe die Abbruchbedingung jetzt bei \(n=1\) angesetzt.

0 Daumen

Nach der ersten Zeile von Tschakabumba geht es wohl so weiter:

\(I_n=\frac{x}{(x^2+1)^n}+2n\cdot\int\frac{x^2}{(x^2+1)^{n+1}}\,dx\)

\(=\frac{x}{(x^2+1)^n}+2n\int(\frac{1}{(x^2+1)^{n}} -\frac{1}{(x^2+1)^{n+1}}   )dx\)

Dann gibt es

$$I_{n+1}=\frac{1}{2n}\left(\frac{x}{(x^2+1)^n}+(2n-1)I_n\right)$$

und weil die ganze Rechnung ( siehe v' in der 1. Zeile) erst für n≥1 gilt,

ist der Rekursionsbegin bei \( I_1 = arctan(x) \).

Avatar von 288 k 🚀
0 Daumen

Hallo :-)

Ich biete folgenden Ansatz an:

$$ \begin{aligned}I_n&=\int\limits \frac{1}{(1+x^2)^n} \text{ dx}=\int\limits \frac{1+x^2-x^2}{(1+x^2)^n} \text{ dx}=\int\limits \frac{1+x^2}{(1+x^2)^n} \text{ dx}+\int\limits \frac{-x^2}{(1+x^2)^n} \text{ dx}\\[15pt]&=\int\limits \frac{1}{(1+x^2)^{n-1}} \text{ dx} -\int\limits \frac{x^2}{(1+x^2)^n} \text{ dx}\\[15pt]&=I_{n-1}-\int\limits \frac{x^2}{(1+x^2)^n} \text{ dx}\end{aligned} $$

Weiter ist:

$$ 1.)\quad \int\limits \frac{x^2}{(1+x^2)^n} \text{ dx}=\int\limits x\cdot \frac{x}{(1+x^2)^n} \text{ dx} $$

Jetzt löse ich:

$$\begin{aligned}2.)\quad &\int\limits \frac{x}{(1+x^2)^n} \text{ dx} \quad \stackrel{z\ \mapsto \ 1+x^2}{ =    }\quad \frac{1}{2}\cdot \int\limits \frac{1}{z^n} \text{ dz}\\[15pt]&=-\frac{1}{2(n-1)}\cdot \frac{1}{z^{n-1}}=-\frac{1}{2(n-1)}\cdot \frac{1}{(1+x^2)^{n-1}}\end{aligned}$$

2.) in 1.) ergibt:

$$ \begin{aligned}&\int\limits \frac{x^2}{(1+x^2)^n} \text{ dx}=\int\limits x\cdot \frac{x}{(1+x^2)^n} \text{ dx}\\[15pt]&=-x\cdot \frac{1}{2(n-1)}\cdot \frac{1}{(1+x^2)^{n-1}}-\int\limits (-1)\cdot \frac{1}{2(n-1)}\cdot \frac{1}{(1+x^2)^{n-1}} \text{ dx}\\[15pt]&=-x\cdot \frac{1}{2(n-1)}\cdot \frac{1}{(1+x^2)^{n-1}}+\frac{1}{2(n-1)}\cdot \int\limits \frac{1}{(1+x^2)^{n-1}} \text{ dx}\\[15pt]&=-x\cdot \frac{1}{2(n-1)}\cdot \frac{1}{(1+x^2)^{n-1}}+\frac{1}{2(n-1)}\cdot I_{n-1}\end{aligned}$$

Das setze ich ins ursprüngliche Integral von oben ein:

$$ \begin{aligned}I_n&=I_{n-1}-\int\limits \frac{x^2}{(1+x^2)^n} \text{ dx}\\[15pt]&=I_{n-1}+x\cdot \frac{1}{2(n-1)}\cdot \frac{1}{(1+x^2)^{n-1}}-\frac{1}{2(n-1)}\cdot I_{n-1}\\[15pt]&=\left(1-\frac{1}{2(n-1)}\right)\cdot I_{n-1}+x\cdot \frac{1}{2(n-1)}\cdot \frac{1}{(1+x^2)^{n-1}},\end{aligned} $$

wobei

$$ I_0=x+c,\quad I_1=\arctan(x)+c' $$

Avatar von 14 k

Ein anderes Problem?

Stell deine Frage

Willkommen bei der Mathelounge! Stell deine Frage einfach und kostenlos

x
Made by a lovely community